LSAT and Law School Admissions Forum

Get expert LSAT preparation and law school admissions advice from PowerScore Test Preparation.

 Administrator
PowerScore Staff
  • PowerScore Staff
  • Posts: 8919
  • Joined: Feb 02, 2011
|
#26242
Complete Question Explanation
(The complete setup for this game can be found here: lsat/viewtopic.php?t=10888)

The correct answer choice is (A)

If only one employee receives a $1k bonus, that employee must be either K or P (L and M are Highly Effective employees and, accordingly, cannot receive $1k bonuses). And, since neither K nor P can receive the highest bonus, one of them must receive a $3k bonus:
June15_game_1_#3_diagram_1.png
Last, we need to ensure that L and M receive higher bonuses than K and P. So, L and M must receive $5k bonuses:
June15_game_1_#3_diagram_2.png
This inference agrees with answer choice (A), which is correct.

Answer choice (B) is not necessarily true, because P could receive a $1k bonus.

Answer choice (C) cannot be true, because M must receive a $5k bonus.

Answer choice (D) is not necessarily true, because K could be the one to receive a $1k bonus.

Answer choice (E) is not necessarily true, because P could be the one to receive a $1k bonus.

Note that K and P are functionally identical: both are employees in the same department, and neither is Highly Effective. There are no rules that apply to one of them but not to the other. So, if answer choice (D) were to be true, then answer choice (E) would have to be true as well (and vice versa), which would be impossible given the unique nature of the correct answer. This line of reasoning is sufficient to justify eliminating answer choices (D) and (E).

Get the most out of your LSAT Prep Plus subscription.

Analyze and track your performance with our Testing and Analytics Package.